Finding Error on Taylor Polynomials using Taylor's Theorem

Click For Summary
Taylor's Theorem can be used to estimate the error in approximating sqrt{8} with a degree 2 Taylor polynomial of f(x)=sqrt{x}, yielding an error estimate of 1/2 * 8^{-7/2}. Additionally, a bound on the difference between sin(x) and its Taylor series expansion x - x^3/6 + x^5/120 for x in the interval [0,1] can be determined using the remainder term from Taylor's Theorem. The discussion emphasizes the importance of applying the error formula for Taylor series to solve these problems. Participants are encouraged to seek further assistance as the problems are not part of a homework assignment and lack provided solutions. Overall, the thread focuses on applying Taylor's Theorem to estimate errors in polynomial approximations.
meichberg92
Messages
1
Reaction score
0
(a) Use Taylor's Theorem to estimate the error in using the Taylor Polynomial of f(x)=sqrt{x} of degree 2 to approximate sqrt{8}. (The answer should be something like 1/2 * 8^{-7/2}.

(b) Find a bound on the difference of sin(x) and x- x^{3}/6 + x^{5}/120 for x in [0,1]This is a problem on a problem sheet that isn't for homework but there are NO solutions. Any help toward a solution would be appreciated.
 
Physics news on Phys.org
meichberg92 said:
(a) Use Taylor's Theorem to estimate the error in using the Taylor Polynomial of f(x)=sqrt{x} of degree 2 to approximate sqrt{8}. (The answer should be something like 1/2 * 8^{-7/2}.

(b) Find a bound on the difference of sin(x) and x- x^{3}/6 + x^{5}/120 for x in [0,1]


This is a problem on a problem sheet that isn't for homework but there are NO solutions. Any help toward a solution would be appreciated.

Use the formula for the error in a Taylor series after the nth term. It is widely available; just Google 'Taylors Theorem'.
 
Question: A clock's minute hand has length 4 and its hour hand has length 3. What is the distance between the tips at the moment when it is increasing most rapidly?(Putnam Exam Question) Answer: Making assumption that both the hands moves at constant angular velocities, the answer is ## \sqrt{7} .## But don't you think this assumption is somewhat doubtful and wrong?

Similar threads

  • · Replies 10 ·
Replies
10
Views
2K
Replies
6
Views
3K
  • · Replies 3 ·
Replies
3
Views
1K
  • · Replies 6 ·
Replies
6
Views
1K
Replies
2
Views
2K
  • · Replies 6 ·
Replies
6
Views
4K
  • · Replies 1 ·
Replies
1
Views
2K
  • · Replies 2 ·
Replies
2
Views
3K
Replies
4
Views
2K
  • · Replies 5 ·
Replies
5
Views
2K